Tuesday, July 24, 2018

Lit Bits: July 24, 2018

From the recent medical literature...

0. Three Free Resources


A. Entertaining: ZDoggMD sings Les Mis

The Confrontation: the spats between emergency physicians and hospitalists can be ... well, misérable.

But ZDoggMD turns these sometimes stressful situations into a musical parody of "The Confrontation" from the hit Broadway musical and movie "Les Misérables." Take a listen!


B. Enlightening: Connections and Curiosity

Winner J. In Practice. A Blog from FPM Journal. Jun 28, 2018

Two ways to find meaning in medicine
Rushing through your workday can leave you feeling frustrated, distracted, and depleted. But mindful interactions can help you feel more effective and fulfilled. To practice mindfulness during patient visits, focus on cultivating connection and curiosity.

1. To build connection, find areas of commonality:

Do you both have children?
Have you both struggled with a parent's illness?
Do you have any similar experiences of suffering, challenges, or joys?

2. To practice curiosity, consider the other person's circumstances:

What is it like to be a migrant farmer?
How does this patient deal with chronic pain or anxiety?
What kind of support does this patient have at home?


C. Educational: SAEM Presentations: Free for the Learning

Up to 225 Hours of Free SAEM18 Content Available on SOAR
   
All SAEM18 educational content is now open access and available online at SOAR (SAEM Online Academic Resources). Experience convenient online and mobile viewing of Advanced EM Workshops, didactics, forums, abstracts—225 hours of original educational content from SAEM18. Downloadable PDFs and MP3 provide convenient, on–the-go viewing, and presenters' slides can be viewed while listening to fully synchronized audio. Enjoy!


1. Recent Acute Stroke Studies

A. Clopidogrel and Aspirin in Acute Ischemic Stroke and High-Risk TIA (the POINT trial).

Johnston SC, et al. N Engl J Med. 2018 Jul 19;379(3):215-225.

BACKGROUND: Combination antiplatelet therapy with clopidogrel and aspirin may reduce the rate of recurrent stroke during the first 3 months after a minor ischemic stroke or transient ischemic attack (TIA). A trial of combination antiplatelet therapy in a Chinese population has shown a reduction in the risk of recurrent stroke. We tested this combination in an international population.

METHODS: In a randomized trial, we assigned patients with minor ischemic stroke or high-risk TIA to receive either clopidogrel at a loading dose of 600 mg on day 1, followed by 75 mg per day, plus aspirin (at a dose of 50 to 325 mg per day) or the same range of doses of aspirin alone. The dose of aspirin in each group was selected by the site investigator. The primary efficacy outcome in a time-to-event analysis was the risk of a composite of major ischemic events, which was defined as ischemic stroke, myocardial infarction, or death from an ischemic vascular event, at 90 days.

RESULTS: A total of 4881 patients were enrolled at 269 international sites. The trial was halted after 84% of the anticipated number of patients had been enrolled because the data and safety monitoring board had determined that the combination of clopidogrel and aspirin was associated with both a lower risk of major ischemic events and a higher risk of major hemorrhage than aspirin alone at 90 days. Major ischemic events occurred in 121 of 2432 patients (5.0%) receiving clopidogrel plus aspirin and in 160 of 2449 patients (6.5%) receiving aspirin plus placebo (hazard ratio, 0.75; 95% confidence interval [CI], 0.59 to 0.95; P=0.02), with most events occurring during the first week after the initial event. Major hemorrhage occurred in 23 patients (0.9%) receiving clopidogrel plus aspirin and in 10 patients (0.4%) receiving aspirin plus placebo (hazard ratio, 2.32; 95% CI, 1.10 to 4.87; P=0.02).

CONCLUSIONS: In patients with minor ischemic stroke or high-risk TIA, those who received a combination of clopidogrel and aspirin had a lower risk of major ischemic events but a higher risk of major hemorrhage at 90 days than those who received aspirin alone.

Editorial excerpt
What is the take-home message for the clinician? The evidence from the SAMMPRIS, CHANCE, and POINT trials is that the combination of aspirin plus clopidogrel reduces the chance of recurrent ischemic stroke during the high-risk period in the first few weeks after a TIA or noncardioembolic ischemic stroke. However, to conform to the results of the POINT trial, if dual therapy is used, it should be confined to the first 3 weeks after a TIA or minor stroke and then transitioned to monotherapy. If patient follow-up and adherence to therapy are not reliable, then dual therapy perhaps should not be considered. Dual therapy may also not be advisable in patients with an uncertain diagnosis of TIA, who either would have been excluded from the trial or did not benefit. Finally, patients who are at increased risk for bleeding, such as those with cerebral microbleeding or a history of brain or systemic bleeding, were excluded from this trial and may not be appropriate candidates for such dual therapy. The POINT trial has provided useful data to help us further personalize our efforts in preventing recurrent stroke.


B. Lytics No Better Than Aspirin in Mild Ischemic Stroke Trial

Effect of Alteplase vs Aspirin on Functional Outcome for Patients With Acute Ischemic Stroke and Minor Nondisabling Neurologic Deficits: The PRISMS RCT

Khatri P, et al. JAMA. 2018;320(2):156-166.

IMPORTANCE:
More than half of patients with acute ischemic stroke have minor neurologic deficits (National Institutes of Health Stroke Scale [NIHSS] score of 0-5) at presentation. Although prior major trials of alteplase included patients with low NIHSS scores, few without clearly disabling deficits were enrolled.

OBJECTIVE:
To evaluate the efficacy and safety of alteplase in patients with NIHSS scores of 0 to 5 whose deficits are not clearly disabling.

DESIGN, SETTING, AND PARTICIPANTS:
The PRISMS trial was designed as a 948-patient, phase 3b, double-blind, double-placebo, multicenter randomized clinical trial of alteplase compared with aspirin for emergent stroke at 75 stroke hospital networks in the United States. Patients with acute ischemic stroke whose deficits were scored as 0 to 5 on the NIHSS and judged not clearly disabling and in whom study treatment could be initiated within 3 hours of onset were eligible and enrolled from May 30, 2014, to December 20, 2016, with final follow-up on March 22, 2017.

INTERVENTIONS:
Participants were randomized to receive intravenous alteplase at the standard dose (0.9 mg/kg) with oral placebo (n = 156) or oral aspirin, 325 mg, with intravenous placebo (n = 157).

MAIN OUTCOMES AND MEASURES:
The primary outcome was the difference in favorable functional outcome, defined as a modified Rankin Scale score of 0 or 1 at 90 days via Cochran-Mantel-Haenszel test stratified by pretreatment NIHSS score, age, and time from onset to treatment. Because of early termination of the trial, prior to unblinding or interim analyses, the plan was revised to examine the risk difference of the primary outcome by a linear model adjusted for the same factors. The primary safety end point was symptomatic intracranial hemorrhage (sICH) within 36 hours of intravenous study treatment.

RESULTS:
Among 313 patients enrolled at 53 stroke networks (mean age, 62 [SD, 13] years; 144 [46%] women; median NIHSS score, 2 [interquartile range {IQR}, 1-3]; median time to treatment, 2.7 hours [IQR, 2.1-2.9]), 281 (89.8%) completed the trial. At 90 days, 122 patients (78.2%) in the alteplase group vs 128 (81.5%) in the aspirin group achieved a favorable outcome (adjusted risk difference, -1.1%; 95% CI, -9.4% to 7.3%). Five alteplase-treated patients (3.2%) vs 0 aspirin-treated patients had sICH (risk difference, 3.3%; 95% CI, 0.8%-7.4%).

CONCLUSIONS AND RELEVANCE:
Among patients with minor nondisabling acute ischemic stroke, treatment with alteplase vs aspirin did not increase the likelihood of favorable functional outcome at 90 days. However, the very early study termination precludes any definitive conclusions, and additional research may be warranted.

Commentary by Ryan Radecki: Stopping the Alteplase Indication Creep

Ever since the narrow approval and strict inclusion criteria of the first trials for alteplase in stroke, our benevolent corporate overlords have been doing their utmost to expand its indications – all while continuing to unilaterally boost its price. This includes sponsoring “expert” convocations to whittle down contraindications, as well as sponsoring, and then cancelling, trials destined to futility.

This is another example of the latter.

This is the remnants of PRISMS, a trial testing the alteplase versus aspirin in a randomized, placebo-controlled trial of mild stroke. In this trial, “mild stroke” included a NIHSS of ≤5 and the absence of any disabling deficits. That is to say, rather, every patient entered in this trial met, in theory, the primary outcome of an mRS of 0-1 at entry. The trial expected to find an advantage to treatment of 9% and incidence of sICH of 2%, a NNT of 11, NNH of 50, and a requirement of 948 patients for the statistical power to validate such findings.

The trial, however, was stopped after 313 patients due to “slow enrollment”. Of these, 32 were lost to follow-up, leaving 281 available for 90-day assessment without imputation. The bulk of patients ranged in NIHSS 1 to 3, with sensory symptoms, facial palsy, and dysarthria the most frequently represented stroke symptoms. Of those with 90-day follow-up, 83.1% of the aspirin arm achieved mRS 0-1, compared with 77.5% of those randomized to alteplase. Conversely, 3.4% of these mild strokes were ultimately mRS 4-6 – a typical definition of “poor outcome” – in the aspirin arm, compared with 10% of those randomized to alteplase. The 5 patients with sICH following alteplase administration contributed to these poor outcomes, compared with none following administration of aspirin.

So, very clearly, there is no evidence here to support a presumption of benefit from alteplase administration, but quite clearly evidence of harm. The authors – with hardly any conflict-of-interest to speak of – go to great lengths to assure us…


C. FAST Fails to Improve TIA, Minor Stroke Response in Britain

Success in boosting major stroke recognition doesn't trickle down to minor symptoms

Medpage. July 2, 2018.

The Face, Arm, Speech, Time (FAST) public education campaign has not improved transient ischemic attack (TIA) and minor stroke response in England, a prospective population-based study found.

In 5 years after the campaign launch, the percentage of people seeking medical attention within 3 hours of a TIA or minor stroke did not change, reported Peter Rothwell, MD, PhD, of John Radcliffe Hospital at University of Oxford, and co-authors.

The percentage of major strokes that were preceded by an unheeded TIA also remained unchanged…



2. Is Inpatient Volume Or ED Crowding A Greater Driver Of Ambulance Diversion?

Hsia RY, et al. Health Affairs 2018 Jul;37(7):1115-1122.

Inpatient volume has long been believed to be a contributing factor to ambulance diversion, which can lead to delayed treatment and poorer outcomes. We examined the extent to which both daily inpatient and emergency department (ED) volumes at specified hospitals, and diversion levels (that is, the number of hours ambulances were diverted on a given day) at their nearest neighboring hospitals, were associated with diversion levels in the period 2005–12. We found that a 10 percent increase in patient volume was associated with a sevenfold greater increase in diversion hours when the volume increase occurred among inpatients (5 percent) versus ED visitors (0.7 percent). When the next-closest ED experienced mild, moderate, or severe diversion, the study hospital’s diversion hours increased by 8 percent, 23 percent, and 44 percent, respectively. These findings suggest that efforts focused on managing inpatient volume and flow might reduce diversion more effectively than interventions focused only on ED dynamics.


3. Cardiogenic Shock and Cardiac Arrest

A. Epinephrine Not Best Choice for Cardiogenic Shock in MI

Associated with higher rates of refractory shock than with norepinephrine

Levy B, et al. Epinephrine Versus Norepinephrine for Cardiogenic Shock After Acute Myocardial Infarction. J Am Coll Cardiol. 2018 Jul 10;72(2):173-182.

BACKGROUND: Vasopressor agents could have certain specific effects in patients with cardiogenic shock (CS) after myocardial infarction, which may influence outcome. Although norepinephrine and epinephrine are currently the most commonly used agents, no randomized trial has compared their effects, and intervention data are lacking.

OBJECTIVES: The goal of this paper was to compare in a prospective, double-blind, multicenter, randomized study, the efficacy and safety of epinephrine and norepinephrine in patients with CS after acute myocardial infarction.

METHODS: The primary efficacy outcome was cardiac index evolution, and the primary safety outcome was the occurrence of refractory CS. Refractory CS was defined as CS with sustained hypotension, end-organ hypoperfusion and hyperlactatemia, and high inotrope and vasopressor doses.

RESULTS: Fifty-seven patients were randomized into 2 study arms, epinephrine and norepinephrine. For the primary efficacy endpoint, cardiac index evolution was similar between the 2 groups (p = 0.43) from baseline (H0) to H72. For the main safety endpoint, the observed higher incidence of refractory shock in the epinephrine group (10 of 27 [37%] vs. norepinephrine 2 of 30 [7%]; p = 0.008) led to early termination of the study. Heart rate increased significantly with epinephrine from H2 to H24 while remaining unchanged with norepinephrine (p less than 0.0001). Several metabolic changes were unfavorable to epinephrine compared with norepinephrine, including an increase in cardiac double product (p = 0.0002) and lactic acidosis from H2 to H24 (p less than 0.0001).

CONCLUSIONS: In patients with CS secondary to acute myocardial infarction, the use of epinephrine compared with norepinephrine was associated with similar effects on arterial pressure and cardiac index and a higher incidence of refractory shock.

B. Epinephrine [Ultimately Not Helpful] in Cardiac Arrest

Adrenaline after cardiac arrest doubles risk of serious brain damage, finds trial.

Perkins GD, et al. A Randomized Trial of Epinephrine in Out-of-Hospital Cardiac Arrest. N Engl J Med. 2018 Jul 18 [Epub ahead of print]

Background Concern about the use of epinephrine as a treatment for out-of-hospital cardiac arrest led the International Liaison Committee on Resuscitation to call for a placebo-controlled trial to determine whether the use of epinephrine is safe and effective in such patients.

Methods In a randomized, double-blind trial involving 8014 patients with out-of-hospital cardiac arrest in the United Kingdom, paramedics at five National Health Service ambulance services administered either parenteral epinephrine (4015 patients) or saline placebo (3999 patients), along with standard care. The primary outcome was the rate of survival at 30 days. Secondary outcomes included the rate of survival until hospital discharge with a favorable neurologic outcome, as indicated by a score of 3 or less on the modified Rankin scale (which ranges from 0 [no symptoms] to 6 [death]).

Results At 30 days, 130 patients (3.2%) in the epinephrine group and 94 (2.4%) in the placebo group were alive (unadjusted odds ratio for survival, 1.39; 95% confidence interval [CI], 1.06 to 1.82; P=0.02). There was no evidence of a significant difference in the proportion of patients who survived until hospital discharge with a favorable neurologic outcome (87 of 4007 patients [2.2%] vs. 74 of 3994 patients [1.9%]; unadjusted odds ratio, 1.18; 95% CI, 0.86 to 1.61). At the time of hospital discharge, severe neurologic impairment (a score of 4 or 5 on the modified Rankin scale) had occurred in more of the survivors in the epinephrine group than in the placebo group (39 of 126 patients [31.0%] vs. 16 of 90 patients [17.8%]).

Conclusions In adults with out-of-hospital cardiac arrest, the use of epinephrine resulted in a significantly higher rate of 30-day survival than the use of placebo, but there was no significant between-group difference in the rate of a favorable neurologic outcome because more survivors had severe neurologic impairment in the epinephrine group.

C. AHA commits to improving in-hospital cardiac arrest survival through innovative competency improvement solution

Maintenance of resuscitation skills a key element in increasing survival rates

AHA. DALLAS, February 24, 2018 — The overall odds of surviving a cardiac arrest in the U.S. could vary by as much as 42 percent between randomly selected hospitals.1  Research has found that poor quality CPR should be considered a preventable harm, and timely delivery of high-quality CPR is the greatest determinant of survival from cardiac arrest.2 Unfortunately, even with trained professionals, poor quality CPR is common.3 In response, the American Heart Association, the world’s leading voluntary health organization devoted to fighting cardiovascular disease and stroke, is addressing the need for alternative training models to support and develop highly skilled healthcare professionals. On February 24, the Association made a commitment to saving an additional 50,000 lives each year at the 6th Annual World Patient Safety, Science & Technology Summit in London, U.K.

The Association is seeking a paradigm shift in hospital resuscitation practice. Advances in technology and evidence now show that low-dose, high-frequency hands on training is significantly more effective than traditional annual or biennial CPR training at increasing and maintaining CPR skills competency. Resuscitation quality improvement initiatives are both necessary and effective at saving and extending lives of in-hospital cardiac arrest patients.4 This shift from a traditional compliance mindset to a resuscitation skills competency approach centered on patient-safety will create a new standard of care, resulting in a transformational, organization-wide resuscitation quality improvement program.

Developed through the Association’s strategic alliance with Laerdal Medical, and launched in February 2015, the Resuscitation Quality Improvement (RQI®) program develops high-quality resuscitation skills through low-dose, high-frequency CPR skills practice, delivering a new standard in resuscitation performance. Validated by actual patient care and survival evidence, lives are being saved.5. Since introduction, approximately 300 hospitals in the U.S. have adopted the Resuscitation Quality Improvement program and the competency of more than 300,000 healthcare providers has been improved.

“Increasing the number of healthcare providers using the Resuscitation Quality Improvement program will save more lives,” says John Meiners, Chief of Mission-aligned Businesses for the American Heart Association. “Approximately 10 minutes of CPR skills practice each quarter helps to eliminate ‘skills decay’ and offer ‘skills mastery,’ resulting in high-quality CPR performance.”

In addition to delivering high-quality CPR, the Resuscitation Quality Improvement program is continuously evolving to integrate additional evidence-based features that can dramatically improve survival as part of a bundle of care. The Resuscitation Quality Improvement Analytics program (currently deployed), combined with structured team debriefings and rapid response teams will help drive excellence in clinical performance.



More on AHA’s Overall Strategy:
Neumar RW, et al. AHA Response to the 2015 Institute of Medicine Report on Strategies to Improve Cardiac Arrest Survival. Circulation. 2018;132:1049-1070.


D. Dual defibrillation in patients with refractory VF: Lit Review

In the setting of cardiac arrest, refractory ventricular fibrillation (VF) is difficult to manage, and mortality rates are high. Double sequential defibrillation (DSD) has been described in the literature as a successful means to terminate this malignant rhythm, after failure of traditional Advanced Cardiac Life Support (ACLS) measures. The authors herein present a case of refractory VF in a patient with cardiac arrest, on whom DSD was successful in reversion to sinus rhythm, and provide a thorough review of similar cases in the literature.


E. Autopsy Recharacterizes Many Cardiac Arrest-Related Deaths

by Ashley Lyles, MedPage Today. June 20, 2018

Many reported sudden cardiac deaths (SCDs) can be attributed to non-sudden causes, with drug overdose and even other organ failures in the top five, according to a study probing the discrepancies between autopsy and SCD surveillance records reported to the medical examiner.

About 40% of such deaths reported from San Francisco County between February 1, 2011 and March 1, 2014 were not unforeseen and were noncardiac, and half were not even arrhythmic, according to a prospective autopsy analysis published in the June 19 issue of Circulation.

Only 55.8% turned out to be autopsy-defined sudden arrhythmic deaths, Zian Tseng, MD, MAS, a cardiac electrophysiologist at the University of California San Francisco, and colleagues reported.

"[T]hese data reflect the decreasing prevalence of coronary disease and increasing prevalence of nonischemic causes; therefore, further inroads into reducing the overall burden of sudden death requires investigation and earlier recognition of nonischemic and nonarrhythmic causes," they wrote.

"We can't assume that all sudden deaths are cardiac," Tseng said in an interview. "I think it's a wake up call for cardiologists overall to say that we can say that death is sudden, but we can't say that it's SCD."

The study's main findings applied to urban and suburban communities alike. On subgroup analysis, the leading causes of death were found to be coronary disease (32%), occult overdose (13.5%), cardiomyopathy (10%), cardiac hypertrophy (8%), and neurological causes (5.5%).

"One of our major findings is that overdose deaths was the second largest cause of death. So these are overdoses that weren't apparent at the scene. These were overdoses that the paramedics called cardiac arrest," Tseng emphasized to MedPage Today. "There was no evidence of drug use at the scene and only after toxicology did we find that these were actually do to overdose, and in more than half of those cases it was due to opioids and so that's very reflective of the ongoing opioid crisis."



4. The Rise of Fake Medical News

More and more websites are peddling “alternative facts” to deceive the public and sell bogus cures. Fixes won’t be simple.

Proto. Mass Gen Hospital. June 18, 2018.

LAST WINTER, THE WEBSITE YourNewsWire published a story with this headline: “CDC Doctor: ‘Disastrous’ Flu Shot Is Causing Deadly Flu Outbreak.” Appearing during one of the worst flu seasons in years, the article quoted an anonymous physician at the Centers for Disease Control and Prevention who warned that nearly all the people dying of the flu had one thing in common: They had gotten flu shots. “This scares the crap out of me,” says the physician in the article. The story also cast Big Pharma as a co- conspirator for failing to disclose to the public the toxic chemicals contained in the vaccine.

None of this was true; the entire story, including the quotes, was fabricated. Yet that didn’t stop the piece from going viral on the internet, popping up on a variety of alternative-health and conspiracy-theory websites. The story was widely shared on Facebook, generating about 500,000 engagements in January alone—more than any story that week from the Wall Street Journal, NPR, ABC, CBS, CNN or Fox News. It also generated thousands of online comments, some fanning broader fears about vaccinations, with “anti-vax” campaigners writing to support the story’s claims and even purported incidents in which the flu shot itself caused paralysis or even death. Although several fact-checking websites poked holes in the story’s narrative, that did nothing to slow its momentum.

Welcome to the world of fake medical news. During and after the 2016 U.S. presidential campaign, the phrase “fake news” marked a new phase of distrust in media, serving on the one hand as a way for politicians to denounce any news coverage they didn’t like, and describing on the other a proliferation of stories that were baldly untrue. The medical realm is not immune and has become home to some of the most egregious examples. “There’s no empirical way to measure it, but my sense is that of all the categories of fake news, medical news is the worst, and there’s more of it out there,” says Kelly McBride, vice president of the Poynter Institute, a nonprofit journalism school in St. Petersburg, Fla.

Watchdog groups have identified hundreds of websites purveying fake medical news, and countless more fly under the radar. This misinformation runs the gamut from truly ridiculous to more subtle misreporting and overhyping of stories from mainstream news sources

The rest of the essay is here (free): http://protomag.com/articles/rise-fake-medical-news?utm

5. Pediatric Corner

A. Should patients with complex febrile seizure be admitted for further management?

Olson H, et al. Am J Emerg Med. 2018;36(8):1386–1390

BACKGROUND AND AIMS: Children with first complex febrile seizure (CFS) are often admitted for observation. The goals of this study were 1) to assess the risk of seizure recurrence during admission, 2) to determine whether early EEG affects acute management.

DESIGN/METHODS: We retrospectively reviewed a cohort of children 6-60months of age admitted from a Pediatric Emergency Department for first CFS over a 15year period. We excluded children admitted for supportive care of their febrile illness. Data extraction included age, gender, seizure features, laboratory and imaging studies, EEG, further seizures during admission, and antiepileptic drugs (AEDs) given.

RESULTS: One hundred eighty three children met inclusion criteria. Seven patients had seizures during the admission (7/183 or 3.8%) Since 38 children were loaded with anti-epileptic medication during their visit, the adjusted rate is 7/145 or 4.8. Increased risk of seizure recurrence during admission was observed in children presenting with multiple seizures (P=0.005). EEG was performed in 104/183 children (57%) and led to change in management in one patient (1%, 95% C.I. 0.05-6%). Six of the 7 children with seizure had an EEG. The study was normal in 3 and findings in the other 2 did not suggest/predict further seizures during the admission.

CONCLUSIONS: Children with first CFSs are at low risk for seizure recurrence during admission. Multiple seizures at presentation are associated with risk of early recurrence and may warrant an admission. EEG had limited effect on acute management and should not be an indication for admission.

B. A Randomized Trial Comparing Metered Dose Inhalers and Breath Actuated Nebulizers.

Snider MA, et al. J Emerg Med. 2018 Jul;55(1):7-14.

BACKGROUND: Despite little evidence for its effectiveness, the breath-actuated nebulizer (BAN) is the default albuterol delivery method in our pediatric emergency department.

OBJECTIVE: We compared the clinical efficacy of BAN and the metered-dose inhaler (MDI) in treating subjects patients 2 to 17 years of age who presented with mild to moderate asthma exacerbations.

METHODS: This is a randomized, nonblinded, noninferiority study conducted at a single pediatric tertiary care emergency department. Subjects presenting with a Pediatric Asthma Score ranging from 5 to 11 received albuterol by BAN or MDI via standard weight-based and symptom severity dosing protocols. Aerosolized ipratropium (via BAN) and intravenous magnesium sulfate were given when clinically indicated. The primary outcome was patient disposition. The noninferiority margin for the primary outcome was an admission rate difference ≤10%. Analyses were adjusted for confounders that were significant at p ≤ 0.10.

RESULTS: We enrolled 890 subjects between October 2014 and April 2015. BAN and MDI groups were comparable for age, gender, and race but not for pretreatment symptom severity; 51% in the MDI group had a Pediatric Asthma Score of moderate severity (8-11) vs. 63% in the BAN group (p less than 0.003). Unadjusted admission rates were 11.9% for MDI and 12.8% for BAN, for an unadjusted risk difference of -0.9% (95% confidence interval -5% to 3%). After adjusting for baseline confounder severity, the risk difference was 2% (95% confidence interval -4% to 7%), which met the criteria for noninferiority.

CONCLUSIONS: Albuterol therapy by MDI is noninferior to BAN for the treatment of mild to moderate asthma exacerbations in children 2 to 17 years of age.

C. Systematic Review: Rectal Administration of Medications for Pediatric Procedural Sedation

Lam SHF, et al. J Emerg Med. 2018 Jul;55(1):51-63.

BACKGROUND: Per rectum (PR) medication delivery is an alternative to traditional oral (PO), intravenous (IV), or intramuscular (IM) administration of medication for procedural sedation of pediatric emergency department patients. However, many emergency physicians are unfamiliar with its use, and there are no widely adopted guidelines or reviews dedicated to this topic.

OBJECTIVE: Our aim was to provide emergency physicians with an overview of PR procedural sedation medications in pediatric patients.

METHODS: We performed a PubMed literature search of relevant keywords limited to studies of human subjects published in English between January 1, 1990 and December 31, 2017. We excluded case reports, general review articles, editorial/opinion pieces, correspondence, and abstracts. Two of the authors then conducted a structured review of the selected studies.

RESULTS: A total of 315 PubMed citations meeting the search criteria were found. Twenty-eight articles were included for final detailed review. Only 4 of the 28 included studies were conducted in the emergency department setting. A total of 9 different medications have been studied for PR procedural sedation. Sedation effectiveness ranged from 40% to 98%. No life-threatening complications were reported in any of the included clinical trials. Hypoxia was found to occur in up to 10% of those receiving PR sedation.

CONCLUSIONS: Pediatric procedural sedation with PR medications appears to be feasible, moderately effective, and safe based on our review of the current literature. However, further studies on its applicability in the emergency department setting are needed.

D. Atypical Presentation of Incomplete Kawasaki Disease: A Peripheral Facial Nerve Palsy.

Rodriguez-Gonzalez M,  et al. J Emerg Med. 2018 Jul;55(1):118-120.

Background
Kawasaki disease (KD) is a multisystem vasculitic disease. Coronary artery aneurysms (CAAs) are the most important and life-threatening complication of KD. Various neurologic complications have been described to occur in 1–30% of patients with KD, but peripheral facial nerve palsy (FNP) is rare (0.9%).

Case Report
We describe a 5-month-old male infant who presented to us with unilateral left infranuclear FNP in the convalescent phase (day 18 of illness) of incomplete KD. The initial diagnosis was not made during the first 10 days of illness (therapeutic window for immunoglobulin treatment) as he was suspected to have hand-mouth-foot disease. We believe that both the delay in diagnosis and treatment of an atypical presentation of KD, combined with the more severe vasculitis and inflammatory burden reported in these cases, contributed to the development of CAA in our patient.

Why Should An Emergency Physician Be Aware of This?
This case highlights the importance of considering KD diagnosis in children with prolonged unexplained fever, even with incomplete diagnostic features, as well as the need to be aware of unusual manifestations, such as FNP. Atypical cases like this may be at increased risk of CAA because of delayed diagnosis and a higher inflammatory burden; therefore, a more aggressive treatment approach may be necessary.

E. Screen time may increase risk of ADHD symptoms

More frequent screen time may boost teens' risk of attention-deficit/hyperactivity disorder symptoms, according to a study of 2,587 10th-graders in Los Angeles in the Journal of the American Medical Association. Findings show that students who were high frequency users of seven or 14 digital media platforms studied were more than twice as likely to develop ADHD symptoms as students who didn't have a high frequency use rate of any of the online social activities.



F. All Young Cannabis Users Face Psychosis Risk

Cannabis use directly increases the risk for psychosis in teens, new research suggests.

Pauline Anderson. Medscape. June 15, 2018

A large prospective study of teens shows that "in adolescents, cannabis use is harmful" with respect to psychosis risk, study author Patricia J. Conrod, PhD, professor of psychiatry, University of Montreal, Canada, told Medscape Medical News.

The effect was observed for the entire cohort. This finding, said Conrod, means that all young cannabis users face psychosis risk, not just those with a family history of schizophrenia or a biological factor that increases their susceptibility to the effects of cannabis.

"The whole population is prone to have this risk," she said.

The study was published online June 6 in JAMA Psychiatry.

Rigorous Causality Test
Increasingly, jurisdictions across North America are moving toward cannabis legalization. In Canada, a marijuana law is set to be implemented later this year.

With such changes, there's a need to understand whether cannabis use has a causal role in the development of psychiatric diseases, such as psychosis.
To date, the evidence with respect to causality has been limited, as studies typically assess psychosis symptoms at only a single follow-up and rely on analytic models that might confound intraindividual processes with initial between-person differences.

Determining causality is especially important during adolescence, a period when both psychosis and cannabis use typically start.

For the study, researchers used random intercept cross-lagged panel models (RI-CLPMs), which Conrod described as "a very novel analytic strategy."

RI-CLPMs use a multilevel approach to test for within-person differences that inform on the extent to which an individual's increase in cannabis use precedes an increase in that individual's psychosis symptoms, and vice versa.

The approach provides the most rigorous test of causal predominance between two outcomes, said Conrod.

"One of the problems in trying to assess a causal relationship between cannabis and mental health outcomes is the chicken or egg issue. Is it that people who are prone to mental health problems are more attracted to cannabis, or is it something about the onset of cannabis use that influences the acceleration of psychosis symptoms?" she said.

The study included 3720 adolescents from the Co-Venture cohort, which represents 76% of all grade 7 students attending 31 secondary schools in the greater Montreal area.

For 4 years, students completed an annual Web-based survey in which they provided self-reports of past-year cannabis use and psychosis symptoms.

Such symptoms were assessed with the Adolescent Psychotic-Like Symptoms Screener; frequency of cannabis use was assessed with a six-point scale (0 indicated never, and 5 indicated every day)…

The remainder of the essay: https://www.medscape.com/viewarticle/898120


6. Brief Summaries from Ann Emerg Med

A. Should Buprenorphine Be Administered to Patients With Opioid Withdrawal in the Emergency Department?

Take-Home Message
Compared with α2-adrenergic agonists (eg, clonidine), buprenorphine improves withdrawal scores and has higher rates of adherence to withdrawal regimen completion.


B. Is Procalcitonin Useful in the Diagnosis and Treatment of Acute Respiratory Infections in the ED?

Take-Home Message
A procalcitonin-guided approach to the initiation and discontinuation of antibiotics in acute respiratory infections has the potential to reduce antibiotic-related adverse effects and duration of therapy, but further study of emergency department (ED) patients is needed.



See also this RCT in a recent issue of NEJM: Procalcitonin-Guided Use of Antibiotics for Lower Respiratory Tract Infection. Abstract: https://www.nejm.org/doi/full/10.1056/NEJMoa1802670

C. What Is the Utility of Noninvasive Ventilation in the Management of Acute Hypercapnic Respiratory Failure Associated With COPD?

Take-Home Message
In patients with acute respiratory failure from chronic obstructive pulmonary disease exacerbation, noninvasive ventilation reduces death and intubation compared with standard treatments.


D. Update: Is Needle Aspiration Better Than Chest Tube Placement for the Management of Primary Spontaneous Pneumothorax?

Take-Home Message
Chest tube placement is successful more often than needle aspiration in the treatment of primary spontaneous pneumothorax, yet is associated with longer hospital length of stay and potentially more adverse events. Therefore, both needle aspiration and chest tube placement are reasonable first-line options, depending on individual patient preferences and circumstances.


E. What Interventions Are Most Effective for Emergency Contraception?

Take-Home Message
For emergency contraception, mifepristone (RU-486) is more effective than levonorgestrel (Plan B), which is more efficacious than an estradiol-levonorgestrel combination (Yuzpe) in preventing pregnancy.


F. Is Low-Dose Ketamine an Effective Alternative to Opioids for the Treatment of Acute Pain in the ED?

Take-Home Message
According to limited evidence, low-dose ketamine and morphine appear to provide similar levels of pain relief at 30 minutes; however, low-dose ketamine is associated with a higher rate of self-limited neuropsychological adverse events.


G. Are Nonsteroidal Anti-inflammatory Drugs Safe and Effective for Treatment of Acute Renal Colic?

Take-Home Message
For patients presenting with acute renal colic, treatment with nonsteroidal anti-inflammatory drugs offers effective pain relief with fewer adverse effects than opioids or paracetamol.


H. Are Corticosteroids Superior to NSAIDs in the Treatment of Acute Gout?

Take-Home Message
Corticosteroids provide pain relief similar to that of nonsteroidal anti-inflammatory drugs for acute gout, with fewer adverse effects.

I. Nerve Blocks: Feeling Blocked? Another Pain Management Tool in the ED


7. First-Ever Ketamine Guidelines for Acute Pain Management Released

Megan Brooks. Medscape. June 15, 2018

Evidence supports the use of intravenous (IV) ketamine for acute pain in a variety of contexts, including as a stand-alone treatment, as an adjunct to opioids, and, to a lesser extent, as an intranasal formulation, according to the first guidelines on the use of ketamine for acute pain management.

Ketamine has captured headlines recently for its potential role in treating severe depression and posttraumatic stress syndrome. Ketamine is also increasingly being used in inpatient and outpatient settings to manage acute pain.

One driving force behind this is the growing effort to reduce the risk for long-term opioid use after acute exposure and its subsequent complications, including addiction. Yet, to date, few recommendations have been available to guide this emerging acute pain therapy.

"The goal of this document is to provide a framework for doctors, for institutions and for payers on use of ketamine for acute pain, who should get it and who should not get it," guideline author Steven Cohen, MD, from Johns Hopkins School of Medicine in Baltimore, Maryland, told Medscape Medical News.

The guidelines were published online June 7 in Regional Anesthesia and Pain Medicine.

The remainder of the essay: https://www.medscape.com/viewarticle/898134


8. Manual Uterine Aspiration: Adding to the Emergency Physician Stabilization Toolkit.

Quinley KE, et al. Ann Emerg Med. 2018 Jul;72(1):86-92. doi:

Emergency physicians are often the first physician contact for women with bleeding from first-trimester miscarriage, which is the most common gynecologic emergency; 1 in 4 women experiences miscarriage in her lifetime.1, 2 Up to 31% of pregnancies end in miscarriage, contributing to the 1.6% (500,000) of emergency department (ED) visits in the United States annually that are prompted by vaginal bleeding in pregnancy.3, 4, 5, 6, 7 Despite the emergency physician’s role as a specialist in hemodynamic stabilization, scant emergency medicine literature addresses the management of patients with hemodynamically significant uterine bleeding in the ED. Manual uterine aspiration, a proven effective, expeditious, and simple technique for uterine evacuation, is a procedure that is routinely taught to obstetric and gynecology residents and midlevel providers, is commonly performed in nonoperative settings, and has classically been excluded from emergency medicine education. The integration of this procedure into the training curriculum of the emergency medicine resident and into the armamentarium of the practicing emergency physician could grant the emergency clinician a stabilizing and potentially lifesaving tool for patients presenting with hemodynamically compromising uterine bleeding from miscarriage.

The idea that emergency medicine residents and physicians should be trained in uterine aspiration for miscarriage is not a novel concept; it was first described in Annals of Emergency Medicine in 1982 and recently discussed in a June 2017 issue of the EM:RAP emergency medicine podcast series.8, 9 We present a brief review of the emergency medicine literature on miscarriage, as well as an overview of the manual uterine aspiration procedure and its indications. We also suggest that emergency medicine residency training programs and attending physicians consider training in manual uterine aspiration, and we explore potential implications for the ED and health care system should ED providers adopt this procedure….


9. Medical Therapy Shows Promise over Surgical Drainage for Peritonsillar Abscess

Ballard DW, et al. Emerg Med News. 2018;40(7):6.

Suction. Check. Head lamp. Check. Cetacaine. Check. Gauze. Check. Vaginal ultrasound probe. Check. Needle and syringe. Check.

Does that sound familiar? This has always been one of the most awkward procedures in the ED: Invasive drainage of a quinsy requires a patient patient, a patient nurse, and, if all goes well, a functional Yankauer. What could go wrong? An overactive gag reflex, a primal scream, or an unexpected intersection with Big Red.

Does it have to be this way, or might there be an opportunity to relegate peritonsillar abscess incision and drainage to the annals of the anachronistic, like plain films for nasal fractures, rectal exams in blunt trauma, and bloodletting for evil humors?

Some limited retrospective literature on the topic suggests that nonoperative management of peritonsillar abscess may already be fairly common and cost-effective and have complications and bounceback rates similar to those of invasive ED management.

The Evidence: Battaglia, et al., studied 12 centers in Southern California over six years (2008-2013) that adopted medical therapy for peritonsillar abscess while seven continued with the surgical drainage and needle aspiration. (Otolaryngol Head Neck Surg 2018;158[2]:280.) Diagnosis was primarily based on clinical exam, with low rates of CT and ultrasound. Medical management consisted of D5NS 1 L bolus; ceftriaxone 1 g and clindamycin 600 mg, both IV; dexamethasone 10 mg IV; and ED observation for one to two hours with PO trial prior to discharge and transition to incision and drainage or needle aspiration if medical management failed. (http://bit.ly/2ItYTnl.) Home management consisted of clindamycin 300 mg QID x 10 days and next-day follow-up with ENT for repeat IV meds and invasive management if the patient had no improvement or worsened.

The authors retrospectively identified 1,787 patients with uncomplicated peritonsillar abscess, and randomly selected 307 (211 receiving medical treatment only) in which to compare outcomes, which included complications, return visit and failure rates, and quality of life outcomes such as volume of opioid prescriptions and number of days feeling sore and days off work. They also performed a sensitivity analysis among those patients reporting trismus to address the potential for selection bias.

The authors reported no difference in complication or failure rates nor any difference in the time until patients could tolerate liquids or solids across cohorts with similar baseline demographic and clinical characteristics. They found, however, that medical management was associated with an eight percent rate of PO challenge failure in the ED, lower volumes of liquid opioid prescribed in the entire study group and trismus-present subgroup, and a lower average of days feeling sore and days off work across all patients. Recognizing that such outcomes could certainly be the result of unaddressed confounding, particularly treatment selection bias, the authors nonetheless concluded that aggressive intervention is not necessary to achieve resolution of pain, resume oral intake, or return to work. We agree with the premise here of less aggressive ED treatment with similar or perhaps even superior results, so let's see how it works in practice….


10. Images in Clinical Practice

The Poor Man's Tox Screen: ECG Findings in the Acute Overdose

Cryptococcus neoformans Meningoencephalitis

Severe Mitral Stenosis

Echinococcal Cysts in the Liver

Vertebral Hydatidosis

Chronic Occlusion of the Superior Vena Cava https://www.nejm.org/doi/full/10.1056/NEJMicm1711273

Tongue Necrosis in Giant-Cell Arteritis

Macular Hole from a Laser Pointer

Nine-Year-Old Girl With Right Hip Pain

Woman With Periorbital Rash

Infant With Poor Feeding

Man With Finger Swelling

Young Woman With Abdominal Pain

Young Adult With Upper Abdominal Pain

Man With Scrotal Mass

Eleven-Year-Old Male With Weakness

A Woman With Severe Rash

Construction Worker With Chest Pain

Young Man With Unilateral Neck Swelling

Young Man With Flank Pain

Girl With Chest Pain

Acute atraumatic pinna (auricular) perichondritis

11. Is hallway care dangerous? An observational study

Rixe JA, et al. Am J Emerg Med. 2018;36(8):1451–1454

OBJECTIVES: In response to crowding the use of hallway beds has become an increasingly prevalent practice in Emergency Departments (EDs). There is limited research on whether caring for patients in hallways (HP) is associated with adverse outcomes. The goal of this study was to examine the effects of HP triage on 30 day outcomes for ED return, readmission, and mortality.

METHODS: We performed a retrospective cohort study at an urban, academic ED comparing HPs (defined as HP for ≥30 min) to matched controls triaged to standard ED beds from 9/30/14 to 10/1/15. We analyzed data from the hospital's clinical data warehouse. Matched controls were selected by gender, age, ethnicity, and language. We used McNemar's test to assess the association between triage location and 30 day study outcomes. We also examined adverse outcomes by triage severity using McNemar's test.

RESULTS: A total of 10,608 HPs were matched to control patients. Compared to controls, HPs had 2.0 times the odds of returning to the ED in 30 days (95% CI: 1.8-2.1), 1.6 times the odds of inpatient readmission (95% CI: 1.4-1.9), and 1.7 times the odds of readmission to observation (95% CI: 1.4-2.0). The odds ratio for mortality in HPs versus controls was 0.80, (95% CI: 0.50-1.3).

CONCLUSIONS: Patients initially triaged to the hallway have an increased odds of 30 day return to the ED, observation and inpatient admission. After adjusting for ESI, the increased odds for return remained similar. The small sample size precluded testing effects of HP status on mortality.

12. AF Management

A. Reduce AF Hospitalizations by Prioritizing ORAL rate reduction meds over IV and Promoting Cardioversion when Applicable

DeMeester S, et al. Implementation of a Novel Algorithm to Decrease Unnecessary Hospitalizations in Patients Presenting to a Community ED With AF. Acad Emerg Med. 2018 Jun;25(6):641-649.

OBJECTIVES: Atrial fibrillation (AFib) is the most common dysrhythmia in the United States. Patients seen in the emergency department (ED) in rapid AFib are often started on intravenous rate-controlling agents and admitted for several days. Although underlying and triggering illnesses must be addressed, AFib, intrinsically, is rarely life-threatening and can often be safely managed in an outpatient setting. At our academic community hospital, we implemented an algorithm to decrease hospital admissions for individuals presenting with a primary diagnosis of AFib. We focused on lenient oral rate control and discharge home. Our study evaluates outcomes after implementation of this algorithm.

METHODS: Study design is a retrospective cohort analysis pre- and postimplementation of the algorithm. The primary outcome was hospital admissions. Secondary outcomes were 3- and 30-day ED visits and any associated hospital admissions. These outcomes were compared before (March 2013-February 2014) and after (March 2015-February 2016) implementation. Chi-square tests and logistic regressions were run to test for significant changes in the three outcome variables.

RESULTS: A total of 1,108 individuals met inclusion criteria with 586 patients in the preimplementation group and 522 in the postimplementation group. Cohorts were broadly comparable in terms of demographics and health histories. Admissions for persons presenting with AFib after implementation decreased significantly (80.4% pre vs. 67.4% post, adjusted odds ratio [OR] = 3.4, p less than 0.001). Despite this difference there was no change in ED return rates within 3 or 30 days (adjusted ORs = 0.93 and 0.89, p = 0.91 and 0.73, respectively).

CONCLUSIONS: Implementation of a novel algorithm to identify and treat low-risk patients with AFib can significantly decrease the rate of hospital admissions without increased ED returns. This simple algorithm could be adopted by other community hospitals and help lower costs.

B. Stroke Prophylaxis After a New ED Diagnosis of AF

Kea B, et al. J Am Coll Cardiol. 2018 Jul 24;72(4):471-472.

Excerpts
Up to 25% of all new atrial fibrillation (AF) diagnoses are made in the emergency department (ED); however, this patient population is often understudied and overlooked. Prior efforts to characterize oral anticoagulation (OAC) prescribing patterns after an ED visit for incident AF is scant, although this information is critical to improve care. Using a national cohort of older U.S. patients, we describe patterns and identify predictors of OAC prescription for patients with a new ED diagnosis of AF.

Medicare fee-for-service OAC-naïve beneficiaries age 65 years and older from 2011 to 2012 who had an incident, new, ED diagnosis of AF (and discharged home) were analyzed… The primary outcome was an OAC prescription by an ED provider, filled after an ED diagnosis of AF, defined as a Part D claim within 10 days of diagnosis. Prescription filling was a proxy for prescribing… A secondary outcome was follow-up care within 30 days of the ED visit, identified from provider and facility outpatient claims. Predictors examined included demographics, comorbidities, prior major bleeding, bleeding risk, and stroke risk scores, and ED and hospital characteristics. An indication for OAC was defined as AF patients with high stroke risk (by CHA2DS2-VASc score of 2 or greater, and low/intermediate bleeding risk (by HAS-BLED score 0-2).

Of the 110,636 beneficiaries with AF, 9,147 met inclusion/exclusion criteria. The mean age was 77.3 (SD 7.9 years); 5,744 (62.8%) were female and 8,488 (92.8%) were Caucasian. Overall, 17.2% patients (1,573 of 9,147) were prescribed an OAC. In the OAC-indicated group, an OAC was prescribed to 18.1% (Table 1). Among OACs prescribed, 62.7% was warfarin followed by direct oral anticoagulant at 26.6%, and combination of warfarin and enoxaparin in 8.3%.

In multivariate analyses, an indication for OAC prescription (high stroke and low/intermediate bleeding risk) was not associated with OAC prescription (aOR: 1.02; 95% CI: 1.00 to 1.04). Presence of heart failure or hypertension (aOR: 1.04; 95% CI: 1.01 to 1.06), diabetes (aOR: 1.03; 95% CI: 1.02 to 1.05), and treatment at teaching hospitals (aOR: 1.05; 95% CI: 1.01 to 1.08) were associated with a higher odds of OAC prescription… More patients who filled an OAC had a cardiology visit within 30 days than those who did not (64.1% vs. 53.7%, p less than 0.001). The proportions of primary care follow-up were similar (45.7% vs. 44.0%, p ¼ 0.220).

In this cohort of Medicare beneficiaries, OAC prescription was infrequent and inconsistent with published professional guidelines… A recent study indicated multiple barriers to U.S. ED provider prescribing (4). A multipronged approach is essential to bridge this gap, including educating EM providers about indications, risks, and benefits of anticoagulation; active multidisciplinary engagement in guideline development and dissemination with emphasis on shared responsibility of stroke prophylaxis; intra-institutional collaboration in development of clinical pathways with short-term follow up that empower ED providers to prescribe OAC for AF.

C. Enlisting EM clinicians to help reduce strokes in high-risk patients with AF and AFL

Vinson DR, et al. Clin Pharmacol Ther. 2018 July 14 [Epub ahead of print]. 

Excerpt
The continued under-prescribing of oral anticoagulants (OACs) described by Hsu and Freeman indicate that improvements in stroke prevention will require concerted efforts from all providers who contribute to AF care. AF patients frequent the ED for rhythm-related complaints, where they often receive rate reduction and cardioversion interventions. Providing this level of care for AF patients creates a natural opportunity for the emergency medicine clinician to discuss stroke prevention with those who are at high risk and are not already on OACs. The ED experience itself might serve as a sentinel event for the AF patient, facilitating needed changes in healthcare behaviors.

There are many ways emergency medicine clinicians can contribute to stroke reduction, ranging from simply introducing the topic with eligible patients to prescribing OACs on discharge (Table). Approximately 20% to 50% of actionable high-risk AF patients receive a prescription of OACs on ED discharge, with higher prescribing for patients who were in the ED primarily for AF-related complaints (4) (see supplemental material). Initiating anticoagulation at the time of discharge for stroke-prone AF patients does not increase bleeding rates and contributes to decreased mortality (2). The most appropriate course of action in each situation, however, will vary by clinician, patient, shift, practice setting, and resource availability. 



12. Physician Cognitive and Interpersonal Performance

A. Influence of Shift Duration on Cognitive Performance of Emergency Physicians: A Prospective Cross-Sectional Study.

Persico N, et al. Ann Emerg Med. 2018;72(2):171–180

STUDY OBJECTIVE:
The relationship between tiredness and medical errors is now commonly accepted. The main objective of this study is to evaluate the cognitive performance of emergency physicians after a night shift of 14 hours (H14) and after a work shift of 24 hours (H24) and to compare it with tests performed after a rest night at home (H0).

METHODS:
Forty emergency physicians (19 staff physicians and 21 residents) were randomly evaluated at H0, H14, and H24. Four cognitive abilities (processing speed, working memory capacity, perceptual reasoning, and cognitive flexibility) were tested according to the Wechsler Adult Intelligence Scale and the Wisconsin Card Sorting Test.

RESULTS:
No cognitive ability was significantly altered after H14 compared with H0. Three of 4 cognitive abilities were impaired at H24 compared with H0: processing speed (11.2 [SD 2.7] versus 12.4 [SD 3.2]; mean difference=-1.2 [95% confidence interval -1.9 to -0.5]), working memory capacity (10.1 [SD 2.9] versus 11.6 [SD 3.0]; mean difference=-1.5 [95% confidence interval -2.2 to -0.8]), and perceptual reasoning (8.4 [SD 2.7] versus 10.6 [SD 2.8]; mean difference=-2.2 [95% confidence interval -3.4 to -1.0]). Cognitive abilities were not different between residents and staff physicians (except for perceptual reasoning) and were not affected by the amount of sleep during the night shift.

CONCLUSION:
The cognitive abilities of emergency physicians were significantly altered after a 24-hour shift, whereas they were not significantly different from the rested condition after a 14-hour night shift. Limiting 24-hour shift work for emergency physicians should be considered and further evaluated.



B. Depression, burnout may contribute to medical mistakes, study finds

Researchers found that physicians who showed signs of burnout or reported having suicidal thoughts in the past year were more than twice as likely to report making medical errors, while those with signs of fatigue were 38% more likely to report making medical errors. The study, reported in Mayo Clinic Proceedings, also showed that pediatricians, anesthesiologists and psychiatrists reported the fewest errors, while radiologists, emergency department physicians and neurosurgeons reported the most.



C. InFocus: How to be a Good Emergency Physician

Roberts JR. Emerg Med News 2018;40(6):8-9.

Excerpt
…Above all, always, always, always be nice. Patients and family rarely remember what you said, but they always remember how you made them feel. There is only one time to make that first impression, a great opportunity to brand yourself as a hero and an angel of mercy. Be nice to the cleaning lady, security guard, cafeteria worker, and x-ray tech. Learn their names; they know yours.

Talk effusively to your patients, talk to them again, and always, always, always talk to the family. Look them in the eye, not into the computer record. Sit down whenever possible; it says you are truly giving them the personal time and attention you would want. That 280-pound demented nursing home patient with bedsores and a feeding tube is somebody's mother, and maybe she was the best third grade teacher Philadelphia ever had.


13. Recent Systematic Reviews from the Emerg Med Literature

A. Safety and Efficacy of IV Lidocaine for Pain Management in the ED: A Systematic Review

Oliveira J E Silva L, et al. Ann Emerg Med. 2018;72(2):135–144.e3.

Study objective
We evaluate the safety and efficacy of intravenous lidocaine in adult patients with acute and chronic pain who are undergoing pain management in the emergency department (ED).

Methods
We searched Ovid CENTRAL, Ovid EMBASE, and Ovid MEDLINE databases for randomized controlled trials and observational studies from inception to January 2017. Efficacy outcomes included reduction in pain scores from baseline to postintervention and need for rescue analgesia. Safety outcomes included incidence of serious (eg, cardiac arrest) and nonserious (eg, dizziness) adverse events. We used the Cochrane Collaboration tool and a modified Newcastle-Ottawa Scale to evaluate the risk of bias across studies. The Grading of Recommendations Assessment, Development and Evaluation approach was used to evaluate the confidence in the evidence available.

Results
From a total of 1,947 titles screened, 61 articles were selected for full-text review. Eight studies met the inclusion criteria and underwent qualitative analysis, including 536 patients. The significant clinical heterogeneity and low quality of studies precluded a meta-analysis. Among the 6 randomized controlled trials included, intravenous lidocaine had efficacy equivalent to that of active controls in 2 studies, and was better than active controls in 2 other studies. In particular, intravenous lidocaine had pain score reduction comparable to or higher than that of intravenous morphine for pain associated with renal colic and critical limb ischemia. Lidocaine did not appear to be effective for migraine headache in 2 studies. There were 20 adverse events reported by 6 studies among 225 patients who received intravenous lidocaine in the ED, 19 nonserious and 1 serious (rate 8.9%, 95% confidence interval 5.5% to 13.4% for any adverse event; and 0.4%, 95% confidence interval 0% to 2.5% for serious adverse events). The confidence in the evidence available for the outcomes evaluated was deemed to be very low because of methodological limitations, including risk of bias, inconsistency, and imprecision.

Conclusion
There is limited current evidence to define the role of intravenous lidocaine as an analgesic for patients with acute renal colic and critical limb ischemia pain in the ED. Its efficacy for other indications has not been adequately tested. The safety of lidocaine for ED pain management has not been adequately examined.

B. PE Among Patients with Acute Exacerbation Of COPD: Implications For EM.

Pourmand A, et al. J Emerg Med. 2018 Jun 23 [Epub ahead of print]

BACKGROUND:
Chronic obstructive pulmonary disease (COPD) is a leading cause of morbidity and mortality worldwide. Common in COPD are acute exacerbations (AE-COPD) that cause acute dyspnea, cough, and bronchospasm. Symptoms of AE-COPD mimic pulmonary embolism (PE).

METHODS:
We conducted a systematic review of the literature to assess the prevalence of PE in patients admitted to the hospital with a clinical diagnosis of AE-COPD. Following the Preferred Reporting Items for Systematic Reviews and Meta-Analysis guidelines, we queried PubMed and MEDLINE databases from 1990 to 2017. The search term "prevalence pulmonary embolism, COPD" was used, and Boolean operators were used to combine search terms. Data were extracted from each article, specifically the sample size, study setting, design, and the prevalence of PE.

RESULTS:
A total of 5 articles were included that demonstrated a prevalence of PE among patients with a clinical diagnosis of AE-COPD that ranged from 3.3-29.1%. Sample sizes varied from 49-197 patients. Studies occurred in both emergency department and inpatient settings, including intensive care units. Among the studies that reported patient characteristics associated with PE in AE-COPD, both obesity and immobility were important.

CONCLUSION:
Studies reporting the prevalence of PE during AE-COPD vary considerably in their methods and results. Because of the relatively high prevalence of PE during AE-COPD, it is important for providers to be aware of this linkage between the 2 conditions and to screen patients using clinical gestalt and validated screening tools until more emergency department data are available.

C. Effectiveness of Implementing Evidence-based Interventions to Reduce C-spine Image Ordering in the ED: A Systematic Review.

Desai S, et al. Acad Emerg Med. 2018 Jun;25(6):672-683.

OBJECTIVES:
Appropriate use of imaging for adult patients with cervical spine (C-spine) injuries in the emergency department (ED) is a longstanding issue. Guidance for C-spine ordering exists; however, the effectiveness of the decision support implementation in the ED is not well studied. This systematic review examines the implementation and effectiveness of evidence-based interventions aimed at reducing C-spine imaging in adults presenting to the ED with neck trauma.

METHODS:
Six electronic databases and the gray literature were searched. Comparative intervention studies were eligible for inclusion. Two independent reviewers screened for study eligibility, study quality, and extracted data. The change in imaging was reported using individual odds ratios (ORs) with 95% confidence intervals (CIs) using random effects.

RESULTS:
A total of 990 unique citations were screened for relevance of which six before-after studies and one randomized controlled trial were included. None of the studies were assessed as high quality. Interventions consisted primarily of locally developed guidelines or established clinical decision rules such as the NEXUS or the Canadian C-spine rule. Overall, implementation of interventions aimed at reducing C-spine image ordering resulted in a statistically significant reduction in imaging (OR = 0.69, 95% CI = 0.51-0.93); however, heterogeneity was high (I2  = 82%). Subgroup analysis revealed no differences between studies that specified enrolling alert and stable patients compared to unspecified trauma (p = 0.81) or between studies employing multifaceted versus nonmultifaceted interventions (p = 0.66). While studies generally provided details on implementation strategies (e.g., teaching sessions, pocket cards, posters, computerized decision support) the effectiveness of these implementation strategies were frequently not reported.

CONCLUSION:
There is moderate evidence regarding the effectiveness of interventions to reduce C-spine image ordering in adult patients seen in the ED with neck trauma. Given the national and international focus on improving appropriateness and reducing unnecessary C-spine imaging through campaigns such as Choosing Wisely, additional interventional research in this field is warranted.


D. The Accuracy and Prognostic Value of Point-of-care US for Nephrolithiasis in the ED: A Systematic Review and Meta-analysis.

Wong C, et al. Acad Emerg Med. 2018 Jun;25(6):684-698.

INTRODUCTION:
Point-of-care ultrasound (POCUS) has been suggested as an initial investigation in the management of renal colic. Our objectives were: 1) to determine the accuracy of POCUS for the diagnosis of nephrolithiasis and 2) to assess its prognostic value in the management of renal colic.

METHODS:
The review protocol was registered to the PROSPERO database (CRD42016035331). An electronic database search of MEDLINE, Embase, and PubMed was conducted utilizing subject headings, keywords, and synonyms that address our research question. Bibliographies of included studies and narrative reviews were manually examined. Studies of adult emergency department patients with renal colic symptoms were included. Any degree of hydronephrosis was considered a positive POCUS finding. Accepted criterion standards were computed tomography evidence of renal stone or hydronephrosis, direct stone visualization, or surgical findings. Screening of abstracts, quality assessment with the QUADAS-2 instrument, and data extraction were performed by two reviewers, with discrepancies resolved by consensus with a third reviewer. Test performance was assessed by pooled sensitivity and specificity, calculated likelihood ratios, and a summary receiver operator curve (SROC). The secondary objective of prognostic value was reported as a narrative summary.

RESULTS:
The electronic search yielded 627 unique titles. After relevance screening, 26 papers underwent full-text review, and nine articles met all inclusion criteria. Of these, five high-quality studies (N = 1,773) were included in the meta-analysis for diagnostic accuracy and the remaining yielded data on prognostic value. The pooled results for sensitivity and specificity were 70.2% (95% confidence interval [CI] = 67.1%-73.2%) and 75.4% (95% CI = 72.5%-78.2%), respectively. The calculated positive and negative likelihood ratios were 2.85 and 0.39. The SROC generated did not show evidence of a threshold effect. Two of the studies in the meta-analysis found that the finding of moderate or greater hydronephrosis yielded a specificity of 94.4% (95% CI = 92.7%-95.8%). Four studies examining prognostic value noted a higher likelihood of a large stone when positive POCUS findings were present. The largest randomized trial showed lower cumulative radiation exposure and no increase in adverse events in those who received POCUS investigation as the initial renal colic investigation.

CONCLUSION:
Point-of-care ultrasound has modest diagnostic accuracy for diagnosing nephrolithiasis. The finding of moderate or severe hydronephrosis is highly specific for the presence of any stone, and the presence of any hydronephrosis is suggestive of a larger (greater than 5 mm) stone in those presenting with renal colic.

E. Effectiveness of Interventions to Decrease Image Ordering for Low Back Pain Presentations in the ED: A Systematic Review

Liu C, et al. Acad Emerg Med. 2018 Jun;25(6):614-626.

BACKGROUND:
Low back pain (LBP) is an extremely frequent reason for patients to present to an emergency department (ED). Despite evidence against the utility of imaging, simple and advanced imaging (i.e., computed tomography [CT], magnetic resonance imaging) for patients with LBP has become increasingly frequent in the ED. The objective of this review was to identify and examine the effectiveness of interventions aimed at reducing image ordering in the ED for LBP patients.

METHODS:
A protocol was developed a priori, following the PRISMA guidelines, and registered with PROSPERO. Six bibliographic databases (including MEDLINE, EMBASE, EBM Reviews, SCOPUS, CINAHL, and Dissertation Abstracts) and the gray literature were searched. Comparative studies assessing interventions that targeted image ordering in the ED for adult patients with LBP were eligible for inclusion. Two reviewers independently screened study eligibility and completed data extraction. Study quality was completed independently by two reviewers using the before-after quality assessment checklist, with a third-party mediator resolving any differences. Due to a limited number of studies and significant heterogeneity, only a descriptive analysis was performed.

RESULTS:
The search yielded 603 unique citations of which a total of five before-after studies were included. Quality assessment identified potential biases relating to comparability between the pre- and postintervention groups, reliable assessment of outcomes, and an overall lack of information on the intervention (i.e., time point, description, intervention data collection). The type of interventions utilized included clinical decision support tools, clinical practice guidelines, a knowledge translation initiative, and multidisciplinary protocols. Overall, four studies reported a decrease in the relative percentage change in imaging in a specific image modality (22.7%-47.4%) following implementation of the interventions; however, one study reported a 35% increase in patient referrals to radiography, while another study reported a subsequent 15.4% increase in referrals to CT and myelography after implementing an intervention which reduced referrals for simple radiography.

DISCUSSION:
While imaging of LBP has been identified as a key area of imaging overuse (e.g., Choosing Wisely recommendation), evidence on interventions to reduce image ordering for ED patients with LBP is sparse. There is some evidence to suggest that interventions can reduce the use of simple imaging in LBP in the ED; however, a shift in imaging modality has also been demonstrated. Additional studies employing higher-quality methods and measuring intervention fidelity are strongly recommended to further explore the potential of ED-based interventions to reduce image ordering for this patient population.

F. Biphasic anaphylaxis: A review of the literature and implications for emergency management.

Pourmand A, et al. Am J Emerg Med. 2018;36(8):1480-5.

BACKGROUND:
The biphasic reaction is a feared complication of anaphylaxis management in the emergency department (ED). The traditional recommended ED observation time is 4-6 h after complete resolution of symptoms for every anaphylaxis patient. However, there has been great controversy regarding whether this standard of care is evidence-based.

METHODS:
Articles were selected using a PubMed, MEDLINE search for the keywords "biphasic anaphylaxis", yielding 155 articles. Articles were filtered by English language, and the keyword biphasic in the title. Case reports and case series were excluded, narrowing to 33 articles. Then, articles were filtered by relevance to the ED setting, and studies conducted in outpatient clinic settings were excluded, narrowing the search to 16 articles. All remaining articles were reviewed and findings were discussed.

RESULTS:
The reported mean time to onset between the resolution of initial anaphylaxis and biphasic reaction ranges widely by study from 1 to 72 h with the majority of studies reporting the mean time to onset over 8 h. A delay between anaphylaxis symptom onset and administration of epinephrine of 60-190 min was reported to correlate with biphasic anaphylaxis in three studies. Anaphylaxis requiring more than 1 dose of epinephrine to achieve symptom resolution was also reported to correlate with biphasic reactions in two studies. No definitive conclusions about the role of corticosteroids in preventing biphasic reactions can be made at this time however; a couple small studies have shown that they may decrease the incidence of biphasic reactions. Additional risk factors correlated with biphasic reaction vary widely between studies and the generalizability of these risk factors is questionable.

CONCLUSIONS:
There is a need for further research to identify true risk factors associated with biphasic anaphylaxis and to clearly define the role of corticosteroids in biphasic reactions. However, given the low incidence and rare mortality of biphasic reactions, patients who receive epinephrine within one hour of symptom onset and who respond to epinephrine with rapid and complete symptom resolution can probably be discharged from the ED with careful return precautions and education without the need for prolonged observation.

Related study: Anaphylaxis-related Malpractice Lawsuits

Lindor RA, et al. West J Emerg Med. 2018 Jul;19(4):693-700.

INTRODUCTION:
Anaphylaxis continues to cause significant morbidity and mortality. Healthcare providers struggle to promptly recognize and appropriately treat anaphylaxis patients. The goal of this study was to characterize anaphylaxis-related malpractice lawsuits.

METHODS:
We collected jury verdicts, settlements, and court opinions regarding alleged medical malpractice involving anaphylaxis from May 2011 through May 2016 from an online legal database (Thomson Reuters Westlaw). Data were abstracted onto a standardized data form.

RESULTS:
We identified 30 anaphylaxis-related malpractice lawsuits. In 80% of cases, the trigger was iatrogenic (40% intravenous [IV] contrast, 33% medications, 7% latex). Sixteen (53%) cases resulted in death, 7 (23%) in permanent cardiac and/or neurologic damage, and 7 (23%) in less severe outcomes. Fourteen (47%) of the lawsuits were related to exposure to a known trigger. Delayed recognition or treatment was cited in 12 (40%) cases and inappropriate IV epinephrine dosing was reported in 5 (17%) cases. Defendants were most commonly physicians (n=15, 50%) and nurses (n=5, 17%). The most common physician specialties named were radiology and primary care (n=3, 10% each), followed by emergency medicine, anesthesiology, and cardiology (n=2, 7% each). Among the 30 cases, 14 (47%) favored the defendant, 8 (37%) resulted in findings of negligence, 3 (10%) cases settled, and 5 (17%) had an unknown legal outcome.

CONCLUSION:
Additional anaphylaxis education, provision of epinephrine autoinjectors or other alternatives to reduce dosing errors, and stronger safeguards to prevent administration of known allergens would all likely reduce anaphylaxis-related patient morbidity and mortality and providers' legal vulnerability to anaphylaxis-related lawsuits.


G. Incidence of intracranial bleeding in anticoagulated patients with minor head injury: a systematic review and meta-analysis of prospective studies.

Minhas H, et al. Br J Haematol. 2018 Jul 20 [Epub ahead of print]

Guidelines advise performing a computed tomography head scan for all anticoagulated head injured patients, but the risk of intracranial haemorrhage (ICH) after a minor head injury is unclear.

We conducted a systematic review and meta-analysis to determine the incidence of ICH in anticoagulated patients presenting with a minor head injury and a Glasgow Coma Score (GCS) of 15. We followed Meta-Analyses and Systematic Reviews of Observational Studies guidelines. We included all prospective studies recruiting consecutive anticoagulated emergency patients presenting with a head injury. Anticoagulation included vitamin-K antagonists (warfarin, fluindione), direct oral anticoagulants (apixaban, rivaroxaban, dabigatran and edoxaban) and low molecular weight heparin.

A total of five studies (including 4080 anticoagulated patients with a GCS of 15) were included in the analysis. The majority of patients took vitamin K antagonists (98·3%). There was significant heterogeneity between studies with regards to mechanism of injury and methods. The random effects pooled incidence of ICH was 8·9% (95% confidence interval 5·0-13·8%).

In conclusion, around 9% of patients on vitamin K antagonists with a minor head injury develop ICH. There is little data on the risk of traumatic intracranial bleeding in patients who have a GSC 15 post-head injury and are prescribed a direct oral anticoagulant.

14. Prospective Validation and Refinement of a Decision Rule to Obtain Chest X-ray in Patients with Nontraumatic Chest Pain in the ED.

Newsom C, et al. Acad Emerg Med. 2018 Jun;25(6):650-656.

OBJECTIVES:
The objective was to prospectively validate and refine previously published criteria to determine the potential utility of chest x-ray (CXR) in the evaluation and management of patients presenting to the emergency department (ED) with nontraumatic chest pain (CP).

What were the criteria for CXR?
We sought to validate Rothrock's high‐yield criteria (age ≥ 60, hemoptysis, prior/current alcohol abuse, prior tuberculosis, prior thromboembolic disease, oxygen saturation less than 90%, respiratory rate over 24, temperature ≥ 100.4°F (38°C), rales, diminished breath sounds) with the addition of the two significant historical variables provided by Hess et al. and Newsom et al. (history of CHF and history of smoking) in a prospective multicenter study.

METHODS:
A prospective observational study was performed of patients presenting to three EDs in the United States with a chief complaint of nontraumatic CP. Previously defined high-risk history and examination elements were combined into a refined decision rule and these elements were recorded for each patient by the ED physician. CXR results were reviewed and analyzed to determine the presence of clinically significant findings including pneumonia, pleural effusion, pneumothorax, congestive heart failure, or the presence of a new mass. Odds ratios for each history and examination element were analyzed as well as sensitivity, specificity, and negative predictive value (NPV) of the rule overall.

RESULTS:
A total of 1,111 patients were enrolled and 1,089 CXRs were analyzed. There were 70 (6.4%) patients with clinically relevant findings on CXR. The refined decision rule had a sensitivity of 92.9% (confidence interval [CI] = 83.4%-97.3%) and specificity of 30.4% (CI = 27.6%-33.4%) to predict clinically relevant findings on CXR, with a NPV of 98.4% (CI = 96.1%-99.4%). Five CXRs with clinically significant findings would have been missed by application of the refined rule (three pneumonias and two pleural effusions). Applying these criteria as a CXR decision rule to this population would have reduced CXR utilization by 28.9%.

CONCLUSIONS:
This study validates previous research suggesting a low clinical yield for CXR in the setting of nontraumatic CP in the ED. This refined clinical decision rule has a favorable sensitivity and NPV in a patient population with low incidence of disease. Further validation is needed prior to use in practice.


15. Patient Preference for Pain Medication in the ED Is Associated with Non-fatal Overdose History.

Whiteside LK, et al. West J Emerg Med. 2018 Jul;19(4):722-730.

INTRODUCTION:
Opioid overdose is a major public health problem. Emergency physicians need information to better assess a patient's risk for overdose or opioid-related harms. The purpose of this study was to determine if patient-reported preference for specific pain medications was associated with a history of lifetime overdose among patients seeking care in the emergency department (ED).

METHODS:
ED patients (18-60 years) completed a screening survey that included questions on overdose history, ED utilization, opioid misuse behaviors as measured by the Current Opioid Misuse Measure (COMM), and analgesic medication preferences for previous ED visits for pain with specific responses for preference for hydromorphone (Dilaudid®), morphine, ketorolac (Toradol®), "no preference" or "never visited the ED for pain." We compared individuals who reported a lifetime history of overdose descriptively to those without a lifetime history of overdose. Logistic regression was used to determine factors associated with a history of overdose.

RESULTS:
We included 2,233 adults in the analysis (71.5% response rate of patients approached) with 532 reporting at least one lifetime overdose. In the univariate analysis, medication preference was significantly associated with overdose history (p less than .001); more patients in the overdose group reported preferring morphine and hydromorphone and those without a history of overdose were more likely to have no preference or say they had never visited the ED for pain. In the logistic regression analysis, patients with higher odds of overdose included those of Caucasian race, participants with a higher COMM score, preference for ketorolac, morphine or hydromorphone. Those who were younger, female and reported never having visited the ED for pain had lower odds of reporting a lifetime overdose. Having "any preference" corresponded to 48% higher odds of lifetime overdose.

CONCLUSION:
Patients with a pain medication preference have higher odds of having a lifetime overdose compared to patients without a specific pain medication preference, even after accounting for level of opioid misuse. This patient-reported preference could cue emergency physicians to identifying high-risk patients for overdose and other substance-related harms.


16. No Radiographic Safe Margin Found in the “Easy IJ” Internal Jugular Vein Procedure

McCarter RN, et al. J Emerg Med. 2018 Jul;55(1):29-33.

BACKGROUND:
The Easy IJ procedure involves placement of a 4.8-cm intravenous catheter into the internal jugular (IJ) vein using ultrasound guidance. It is not known whether this needle length has the potential to cause a pneumothorax.

OBJECTIVE:
The objective of this study was to determine if a radiographic "safe margin" exists. We hypothesized that an average margin of ≥2 cm would exist between the catheter tip and the pleura.

METHODS:
Operators used a central approach to the IJ vein. We reviewed radiographic images taken immediately after the Easy IJ procedure. Using digital software, we measured the distance from the catheter tip to the closest point of the pleura and from the catheter tip to the level of the lung apex. We defined distances exceeding the margin of safety-either passing the pleura or ending inferior to the apex-as negative for the purpose of calculating an average. We used the t distribution to calculate 95% confidence intervals (CIs) for average values.

RESULTS:
Radiographs showing the catheter tip were available from 62 patients. The mean needle-to-pleura distance was -0.1 cm (95% CI -0.7 to 0.5 cm). The mean vertical distance to the apex was -0.2 cm (95% CI -0.8 to 0.3 cm), with a standard deviation of 2.25 cm.

CONCLUSION:
Radiographic analysis failed to show a margin of safety for the Easy IJ procedure. Postprocedure imaging may still be necessary to exclude pneumothorax.

17. The Newest Threat to ED Procedural Sedation.

Green SM, et al. Ann Emerg Med. 2018

Article Outline
What Happened to Deep Sedation?
An Anesthesiologist Guideline to Govern All Specialties?
Propofol and Ketamine: “Medications Intended for General Anesthesia”

Excerpt
Now the ASA has released updated sedation guidelines that again assert a scope beyond the practice of anesthesiologists, stating that their guidelines “are intended for use by all providers…in any inpatient or outpatient setting.”11 These new guidelines contain vague, confusing, and misleading statements that run contrary to the existing scientific evidence and threaten the well-established sedation practices of emergency physicians and other specialists. Procedural sedation has long been a core competency in emergency medicine and critical care medicine, and our patients depend on us to provide effective sedation and analgesia for procedures that are often extremely painful (eg, cardioversion, abscess incision and drainage, fracture and dislocation reduction) or unduly frightening (eg, facial laceration repair, neuroimaging in a child). These revised ASA guidelines restrict the use of propofol and ketamine—our 2 most commonly administered sedative agents12, 13, 14, 15, 16, 17—and any adoption or enforcement of these directives would restrict emergency physician access to these drugs, resulting in widespread use of alternative agents that are less safe and provide much less effective sedation and analgesia.

Despite these concerns, there are some positive aspects to this update…


Related
ACEP Hosts Meeting to Create Unscheduled-Sedation Guidelines


18. High-Velocity Nasal Insufflation in the Treatment of Respiratory Failure: A Randomized Clinical Trial.

Doshi P, et al. Ann Emerg Med. 2018 Jul;72(1):73-83.e5. doi:

STUDY OBJECTIVE: We compare high-velocity nasal insufflation, a form of high-flow nasal cannula, with noninvasive positive-pressure ventilation in the treatment of undifferentiated respiratory failure with respect to therapy failure, as indicated by requirement for endotracheal intubation or cross over to the alternative therapy.

METHODS: This was a multicenter, randomized trial of adults presenting to the emergency department (ED) with respiratory failure requiring noninvasive positive-pressure ventilation. Patients were randomly assigned to high-velocity nasal insufflation (initial flow 35 L/min; temperature 35°C (95°F) to 37°C (98.6°F); FiO2 1.0) or noninvasive positive-pressure ventilation using an oronasal mask (inspiratory positive airway pressure 10 cm H2O; expiratory positive airway pressure 5 cm H2O). The primary outcome was therapy failure at 72 hours after enrollment. A subjective outcome of crossover was allowed as a risk mitigation to support deferment of informed consent. Noninferiority margins were set at 15 and 20 percentage points, respectively.

RESULTS: A total of 204 patients were enrolled and included in the analysis, randomized to high-velocity nasal insufflation (104) and noninvasive positive-pressure ventilation (100). The intubation rate (high-velocity nasal insufflation=7%; noninvasive positive-pressure ventilation=13%; risk difference=-6%; 95% confidence interval -14% to 2%) and any failure of the assigned arm (high-velocity nasal insufflation=26%; noninvasive positive-pressure ventilation=17%; risk difference 9%; confidence interval -2% to 20%) at 72 hours met noninferiority. The effect on PCO2 over time was similar in the entire study population and in patients with baseline hypercapnia. Vital signs and blood gas analyses improved similarly over time. The primary limitation was the technical inability to blind the clinical team.

CONCLUSION: High-velocity nasal insufflation is noninferior to noninvasive positive-pressure ventilation for the treatment of undifferentiated respiratory failure in adult patients presenting to the ED.

19. Necrotizing Fasciitis Diagnoses and Therapy Tips

By Anton Helman, MD, CCFP(EM), FCFP

How to quickly recognize and begin treating this rapidly progressing infection of the fascia with secondary necrosis of the subcutaneous tissues.

Necrotizing fasciitis is a rapidly progressive inflammatory infection of the fascia, with secondary necrosis of the subcutaneous tissues. The spectrum of presentation is wide, ranging from a benign-appearing rash in a well person to obvious skin necrosis with hemodynamic instability, multi-organ failure, and death. Patients who present early in this spectrum of disease are difficult to diagnose, with an initial misdiagnosis rate of 71.4 percent.1 However, initiation of treatment in these early stages gives patients the greatest chance of survival in this otherwise deadly disease. In this early phase, necrotizing fasciitis can be mistaken for simple cellulitis, and while the skin may appear benign, it is often the tip of the iceberg to what lies beneath.

The diagnostic difficulty also lies in the fact that there are no lab test results or even imaging that can definitively rule out necrotizing fasciitis. In fact, the diagnosis is a clinical one that can only be confirmed with surgical exploration. Therefore, it is imperative that if you have anything more than the slightest suspicion based on your clinical exam, you consider early consultation with a surgeon for definitive diagnosis and surgical debridement as well as start empiric antibiotics. Lab findings that are suggestive but not diagnostic of necrotizing fasciitis include coagulopathy, hypoalbuminemia, thrombocytopenia, lactic acidosis, creatine phosphokinase elevation, and C-reactive protein (CRP) elevation, which all tend to occur in later stages of disease.1

While clinical decision tools, such as the Laboratory Risk Indicator for Necrotizing Fasciitis (LRINEC) score that includes CRP, white blood cell count, hemoglobin, sodium, creatinine, and glucose, might help raise your suspicion for necrotizing fasciitis, validation studies showed that a LRINEC cutoff of six points only had a negative predictive value of 92.5 percent.2,3 While these lab findings and imaging findings of subcutaneous air and fascial thickening on X-ray, CT, and MRI can help support the diagnosis, they should not delay definitive treatment in the operating room in clinically obvious cases and should never override clinical judgment.

Findings on point-of-care ultrasound, which has the advantage of speed over other imaging modalities, may help support the diagnosis but again cannot rule it out.4……


20. A Simple ED Intervention Can Help Cut Suicide Risk

Chatterjee. NPR. July 11, 2018

Many people who attempt suicide end up in an emergency room for immediate treatment. But few of those suicide survivors get the follow-up care they need at a time when they are especially likely to attempt suicide again.

Now, a study shows that a simple intervention conducted by staff in emergency departments can reduce the risk of future attempts. The intervention involves creating a safety plan for each patient and following up with phone calls after discharge.

"It reduced the odds of suicidal behavior by half," says Barbara Stanley, a psychologist at Columbia University and the lead author of the study. "That's a phenomenal difference."

The study, which was published Wednesday in JAMA Psychiatry, included 1,200 patients at five Veterans Affairs hospitals around the country. The findings offer a way for hospitals and clinics to help reduce the rising numbers of death by suicide across the country.

“When someone goes to the emergency room for a suicide attempt, they are at risk of another suicide attempt for the next three months," says Stanley. "It's like a ticking time bomb."

This is the first large-scale study of the Safety Planning Intervention, which Stanley and her group developed in 2008 and which has been adopted at hospitals and clinics around the country. It can be provided by a physician, a nurse or a social worker and requires very little training….



21. Only 30% of patients given follow-up medications after opioid overdose

A study in the Annals of Internal Medicine found that only three in 10 patients who survived an opioid overdose were prescribed some type of medication-assisted treatment that could prevent a subsequent life-threatening event. The findings, based on data from 17,568 people in Massachusetts who overdosed on opioids, showed 3,022 received buprenorphine and 2,040 received methadone, and the groups had a 40% and 60% lower death rate after one year, respectively, compared with patients who were not prescribed any medications.


22. Prevalence and outcomes of incidentalomas on imaging: umbrella review

O’Sullivan JW, et al. Cite this as: BMJ 2018;361:k2387

Introduction
Incidentalomas, incidental imaging findings serendipitously diagnosed in an asymptomatic patient or symptomatic patient undergoing imaging for an unrelated reason,123 are fast becoming a modern medical crisis.4 The rapid rise in demand for imaging,56 coupled with rapidly advancing image resolution is driving a surge in incidentalomas.789

Incidentalomas are problematic because they can lead to patient anxiety.10 Given the clinical uncertainty surrounding their ideal management, they can encourage further investigations and potentially overtreatment. In response to the risks of incidentalomas, a US Presidential Commission encouraged clinicians to discuss with patients incidentalomas as a risk of imaging before they order a scan.4 Despite this, informed consent before imaging remains poor.11 It is difficult for clinicians to appropriately inform patients of the chance of incidental findings because the data quantifying prevalence is inconsistent and unclear.

Clinicians have also expressed uncertainty about the ideal management of incidentalomas, particularly primary care physicians.312 Although some guidelines exist,11314151617 they are often directed at radiologists151617 and for many organs (eg, colon, spine, breast, brain, prostate, and parotid) there are no guidelines to address the management of incidentalomas…
The rest of the study (full-text free): https://www.bmj.com/content/361/bmj.k2387

23. The New Alternatives to the ConCert Examination: Will the Shorter Tests Make Emergency Physicians Less Miserable?

Kelly M. Ann Emerg Med. 2018;72:A15–A17.

Excerpt
Beyond the cost in terms of money, anxiety, and time, however, some emergency physicians have a more significant complaint: They grouse that the recertification process has done nothing to add to their hands-on skills or help their patients. “None of us can think of anything more irrelevant to the practice of emergency medicine than taking an outdated multiple-choice test every 10 years,” said Paul Tripathi, MD, the lead physician at Inova Fair Oaks Hospital pediatric emergency department. He remarked that the test in no way improved him as a physician during the 9 years when he wasn’t studying for it—and also didn’t help once he passed because of the “irrelevant and non–real-world material.”


24. Micro Bits

A. Complementary medicine tied to lower cancer survival rates

The use of complementary medicine by cancer patients was associated with poorer overall five-year survival and an increased mortality risk, compared with not using CM, researchers reported in JAMA Oncology. The increase in mortality was attributed to patients delaying or refusing potentially curative conventional treatments.

Medscape (registration required): https://www.medscape.com/viewarticle/899554


B. Common symptoms before multiple sclerosis diagnosis identified

A Canadian study in the Multiple Sclerosis Journal found common symptoms of multiple sclerosis five years before diagnosis include mental health disorders, pain and sleep problems. The findings, based on health records for 14,000 patients with multiple sclerosis and 67,000 controls without the disease, also showed that those who were later diagnosed with the disease more commonly have fibromyalgia, irritable bowel syndrome and migraines, compared with controls.

C. Study links childhood socioeconomic disadvantages to health risks in adulthood

Researchers followed 3,467 individuals in Finland for an average of 30 years and found that growing up in a socioeconomically disadvantaged neighborhood was associated with increased risk of type 2 diabetes, obesity, hypertension and fatty liver in adulthood. The findings in The Lancet Public Health "support policies that increase resources and opportunities for those living in socioeconomically disadvantaged areas," researchers said.

D. Risk For Heart Attack During Pregnancy Increasing In US Women, Research Indicates.

Research indicates “the number of women who have a heart attack during pregnancy, labor or in the weeks following birth appears to be rising.” The findings were published in Mayo Clinical Proceedings.


E. FDA Updates Safety Warnings for Fluoroquinolones

July 17, 2018 04:18 pm News Staff – Following an extensive review of adverse event reports and case reports published in the medical literature, the FDA on July 10 announced changes(www.fda.gov) to the safety labeling of fluoroquinolones. The changes include strengthened warnings about the risks of mental health side effects and disturbances in blood sugar, and more consistent labeling for all fluoroquinolones administered orally or by injection.


F. Taking NSAIDs with anticoagulants may raise risk for bleeding, stroke

Patients with atrial fibrillation who use oral anticoagulants and nonsteroidal anti-inflammatory drugs are at an increased risk for major bleeding and stroke, based on findings from the Randomized Evaluation of Long-Term Anticoagulant Therapy trial reported in the Journal of the American College of Cardiology. "This study shows that both with warfarin and the newer anticoagulant dabigatran, you get more bleeding, but you can also get more thrombotic complications when you add an NSAID," according to Sam Schulman, MD, who co-wrote an accompanying editorial on the study.


G. Pills are not the answer to unhealthy lifestyles

More than half of adults aged over 45 will be labelled as hypertensive if new US guidelines are adopted, concludes a study in The BMJ this week (doi:10.1136/bmj.k2357). This equates to 70 million people in the US and 267 million people in China being eligible for antihypertensive drugs, a marked increase on already high rates of drug treatment for high blood pressure.


H. Most patients get 11 seconds to explain symptoms before interruption

Researchers who analyzed recorded consultations between 112 patients and their doctors found that in 7 out of every 10 cases, physicians interrupted patients as they explained their symptoms within a median 11 seconds. The study, in the Journal of General Internal Medicine, found primary care physicians allowed patients more time to describe their ailments without interruption than specialists.


I. Review: No evidence of heart health benefits with vitamin-mineral supplements

A review of 18 studies with more than 2 million participants showed that the use of multivitamins and mineral supplements was not associated with the risk of death from coronary heart or cardiovascular disease, or of stroke incidence or deaths. The review was published in Circulation: Cardiovascular Quality and Outcomes.


J. Early solid food introduction may improve infant sleep

Breastfed babies who ate solid foods at age 3 months had almost two hours of additional sleep and nearly two fewer sleep disturbances per week, with sleep improvements persisting even after the first year of infancy, compared with those who were exclusively breastfed until age 6 months, UK researchers reported in JAMA Pediatrics. The findings also showed that very serious sleep problems were less likely to be reported by parents of those in the early solid food introduction group.


K. Will You Forgive Me for Saving You?

Excerpt
I hope you forgive me. You were so close to gone, and I was so afraid you wouldn’t make it. Even then, I knew that “making it” would be relative, and the life I was saving you for would be troubled at best. I hope some part of you finds joy and that it overshadows the pain. I hope you remember a little, because I will never forget you.

I wonder: Did I save you for a good life? Are you glad I did it? Will your mom and grandma still thank me when the endless days of caring for you heap into years of sacrifice? Will you forgive me for saving you?


L. Weight loss tied to atrial fibrillation reversal, study finds

Australian researchers used a cohort of 355 patients with atrial fibrillation and obesity and found more of those who lost at least 10% of their weight experienced a reverse progression of AF after four years, compared with those who lost less than 3% of their weight. The findings in the journal Europace also revealed that 88% of those who lost at least 10% of their weight did not have AFib in the past year or had gone from persistent AFib to paroxysmal AFib episodes.


M. Atul Gawande Named CEO of Health Venture by Amazon, Berkshire Hathaway and JPMorgan


N. How 3 big Southern California hospitals are dealing with their growing number of ER visitors

Kaiser, USC, and Antelope Valley


O. A little satirical reprieve

Five-Second Rule Validated in the Operating Room: 'Total game changer'

Skelepuns. June 21, 2018. CHICAGO – In a paradigm shift for the operating room, a new study has found that reusing instruments dropped on the operating room floor does not increase the incidence of postoperative infections, so long as the instruments are picked up within 5 seconds of being dropped.

In this multi-center, prospective, randomized, double-blind placebo-controlled study, surgical instruments were dropped on the operating room floor and picked up immediately and at 5, 10, and 15 seconds and then used in surgery. It was found that using instruments dropped for 5 seconds or less did not result in an increased incidence of infections. However, using instruments dropped for more than 5 seconds resulted in overwhelming sepsis in 100% of study patients.

"This is a total game changer for the OR. No more opening peel packs when the medical student drops the suture scissors and then drops the replacement suture scissors. We just have to make sure that anesthesia actually gets the antibiotics in before incision," said lead study author Sharon Weeksie. "Sure, you drop them and they get dirty. But it's a distinction without a difference."

Blowing on the instruments after pickup was not found to affect infection rates in any meaningful way.